Club CIE

2

Réponses

  • Quentino37 personnellement je n'ai pas réfléchi à ton intégrale, ce n'est pas dans le style de ce fil
    Le 😄 Farceur


  • Quentino37
    J'ai une de spéciale pour toi
    8) Montrer que $$\int_{0}^{\frac{1}{2}}{\Gamma \left( 1+x \right)\Gamma \left( 1-x \right)dx}=\frac{2G}{\pi }$$
    Le 😄 Farceur


  • à... pourtant elle calcule :-)
    PS: j'ai encore envoyé un message en même temps que toi
    Je suis donc je pense 
  • merci gebrane!
    Je suis donc je pense 
  • L'égalité $\displaystyle \frac{1}{2} \text{Li}_2(x^2) = \text{Li}_2(x) + \text{Li}_2(-x)$ pour $-1\leq x\leq 1$ est évidente si on se rappelle la définition de $\text{Li}_2$ sous forme de série*.

    *$-1\leq x\leq 1,\displaystyle \text{Li}_2(x)=\sum_{n=1}^\infty \frac{x^n}{n^2}$
  • \[\int_{0}^\frac{1}{2}\Gamma(1-x)\Gamma(1+x)dx=\int_{0}^\frac{1}{2}x\Gamma(1-x)\Gamma(x)dx=\int_{0}^{\frac{1}{2}}\frac{\pi x}{\sin(\pi x)}dx=\frac{1}{\pi}\int_{0}^{\frac{\pi}{2}}\frac{t}{\sin(t)}dt\\
    G=\frac{1}{2}\int_{0}^{\frac{\pi}{2}}\frac{t}{\sin(t)}dt\\
    \frac{1}{\pi}\int_{0}^{\frac{\pi}{2}}\frac{t}{\sin(t)}dt=\frac{2G}{\pi}.
    \] Et voila c'est prouvé (tu)
    Je suis donc je pense 
  • rien à dire ( quand tu sera grand et célèbre, n'oublie pas ce petit gebrane à capacités modestes )
    Le 😄 Farceur


  • Quentino37.
    Pourquoi $\displaystyle \text{G}=\frac{1}{2}\int_{0}^{\frac{\pi}{2}}\frac{t}{\sin(t)}dt$ ?
  • FDP on devient vieux B-)- ?
    Le 😄 Farceur


  • à Fin de Partie: en effet.
    A demon  wind propelled me east of the sun
  • gilles benson tu cliques sur le smiley qui fume dans mon message et tu comprendras
    FDP oublie vite
    Le 😄 Farceur


  • Bonjour

    Que vaut $ I=\displaystyle \int_{2+\sqrt{3}}^\infty \dfrac{\ln t}{1+t^2} dt $
     
  • Gebrane: La question n'était pas pour moi. Ce que je voulais dire est que, c'est admissible dans le contexte qu'on utilise sans démonstration la formule des compléments de la fonction gamma mais balancer la valeur d'une intégrale comme étant évidente c'est autre chose.
    (j'ai passé pas mal de temps à trouver une démonstration, que personne ne me demandait, de la valeur de $\text{Li}_2\left(\frac{1}{\varphi}\right)$ pour éviter ce travers)
  • bonjour, le "en effet" répondait à la remarque concernant l'identité $ Li_2(x) + Li_2(-x)$. En ce qui concerne $Li_2(1/\phi)$, je veux bien croire que celà ait pris du temps à Fdp puisqu'il a retrouvé à la main tous les ingrédients de la méthode trouvée dans Lewin et il y a bien une différence entre utiliser certains résultats qui ne sont pas évidents mais monnaie courante et l'obscure valeur d'une fonction peu utilisée en un point aussi particulier que $1/\phi$.
    A demon  wind propelled me east of the sun
  • Je commence à avoir un peu d'expérience dans ce genre de calculs. Les ingrédients sont toujours les mêmes: changements de variable de type $y=\dfrac{a+bx}{a'+b'x}$ ($a,b,a',b'$ entiers) bien choisis et intégration par parties.
    La difficulté est de faire "le" bon enchaînement.
    Dans le cas d'espèce, la première idée est d'essayer de trouver un changement de variable (comme ceux mentionnés plus haut) qui va ramener à l'intervalle $[0;1]$ pour faire disparaître le $\varphi$.
    Mais on ne va pas y arriver. Le mieux qu'on puisse faire est $y=\dfrac{1}{x(1+x)}$.
    Donc il ne reste plus qu'à trouver les changements de variable qui laissent invariant l'intervalle $\left[0,\dfrac{1}{\varphi}\right]$ et espérer après calcul se retrouver avec une égalité du type $J=a+bJ$ avec $a,b$ des constantes, $b$ n'est pas égal à $1$.
  • Et pour $\text{Li}_2 (\phi)$ ?
    Le 😄 Farceur


  • Lewin indique que le résultat n'était pas connu en 1958...
    A demon  wind propelled me east of the sun
  • Maintenant en 2021 c'est connu ?
    Le 😄 Farceur


  • On connaît des valeurs pour les images de $1/\phi$, $1/\phi ^2$, $-1/\phi$ et $-\phi$.
    A demon  wind propelled me east of the sun
  • bonsoir, j'ai regardé en béotien $I = \displaystyle \int_{2+\sqrt{3} }^ {+\infty} \dfrac{\ln t}{1+t^2} \mathrm dt$; on sait que si $a \geq 0$, on a $\displaystyle \int_{1/a }^ {a} \dfrac{\ln t}{1+t^2} \mathrm dt = 0$ en posant $ u = 1/t$ et ceci vaut pour $a = +\infty$; l'intégrale vaut donc $I = - \displaystyle \int_0^{2-\sqrt{3} } \dfrac{\ln t}{1+t^2} \mathrm dt$ et on peut développer en série.
    A demon  wind propelled me east of the sun
  • Bonjour,

    @bd2017 : $\displaystyle \int_{2+\sqrt{3}}^{+\infty} { \ln t \over 1+t^2} dt = {2 \over 3}G$ avec $G$ la constante de Catalan.
  • Calculer

    $$\int_{0}^{+oo}\frac{\arctan(x)}{x^5+1} dx$$
  • Bonjour, mathematica, donne: $\pi/12 \ln (a) + 6i \left( Li_2 ( i a) -Li_2 (-i a) \right)$ avec $a = 2 - \sqrt{3}$.

    Moi, je suis arrivé à $\pi/12 \ln (a) - \displaystyle \sum_{n\geq0} (-1)^n \dfrac{a^{2n+1}}{(2n+1)^2}$ vu que $\arctan (a) = \pi/12$ et que $ 2 - \sqrt{3}$ et $ 2 + \sqrt{3}$ sont inverses l'un de l'autre.
    A demon  wind propelled me east of the sun
  • Gilles Benson:: Je suis curieux de voir comment tu obtiens le résultat en développant en série? B-)-

    \begin{align}I&= - \displaystyle \int_0^{2-\sqrt{3} } \dfrac{\ln t}{1+t^2} \mathrm dt\\
    &\overset{t=\tan x}=-\int_0^{\frac{\pi}{12}}\ln(\tan x)dx\\
    &=-2\int_0^{\frac{\pi}{12}}\ln(\tan(3x))dx\tag1\\
    &\overset{y=3x}=-\frac{2}{3}\int_0^{\frac{\pi}{4}}\ln(\tan y)dy\\
    &=\frac{2}{3}\text{G}
    \end{align}

    Preuve de $(1)$:
    Pour tout $\displaystyle x\in\left[0,\frac{\pi}{12}\right],\tan(3x)=\tan x\tan\left(\frac{\pi}{3}-x\right)\tan\left(\frac{\pi}{3}+x\right)$
    \begin{align}\int_0^{\frac{\pi}{12}}\ln(\tan(3x))dx&=\int_0^{\frac{\pi}{12}}\ln(\tan x)dx+\underbrace{\int_0^{\frac{\pi}{12}}\ln\left((\tan\left(\frac{\pi}{3}-x\right)\right)dx}_{y=\frac{\pi}{3}-x}+\underbrace{\int_0^{\frac{\pi}{12}}\ln\left(\tan\left(\frac{\pi}{3}+x\right)\right)dx}_{y=\frac{\pi}{3}+x}\\
    &=\int_0^{\frac{\pi}{12}}\ln(\tan x)dx+\int_{\frac{\pi}{4}}^{\frac{\pi}{3}}\ln(\tan y)dy+\int_{\frac{\pi}{3}}^{\frac{5\pi}{12}}\ln(\tan y)dy\\
    &=\int_0^{\frac{\pi}{12}}\ln(\tan x)dx+\underbrace{\int_{\frac{\pi}{4}}^{\frac{5\pi}{12}}\ln(\tan y)dy}_{z=\frac{\pi}{2}-y}\\
    &=\int_0^{\frac{\pi}{12}}\ln(\tan x)dx-\int_{\frac{\pi}{12}}^{\frac{\pi}{4}}\ln(\tan z)dz\\
    &=2\int_0^{\frac{\pi}{12}}\ln(\tan x)dx-\underbrace{\int_0^{\frac{\pi}{4}}\ln(\tan z)dz}_{u=\frac{1}{3}z}\\
    &=2\int_0^{\frac{\pi}{12}}\ln(\tan x)dx-3\int_0^{\frac{\pi}{12}}\ln(\tan(3u))du\\
    &\boxed{\int_0^{\frac{\pi}{12}}\ln(\tan x)dx=2\int_0^{\frac{\pi}{12}}\ln(\tan(3u))du}
    \end{align}

    La preuve de $(1)$ vient de https://citeseerx.ist.psu.edu/viewdoc/download;jsessionid=BB5B0A4CEC3F2559A42D631E60589C30?doi=10.1.1.26.1879&rep=rep1&type=pdf
    (égalité numéro 31)
  • On peut aussi faire:
    Considérer,
    \begin{align*}
    K&=\int_0^1 \frac{\arctan\left(\frac{x}{1-x^2}\right)}{x}\,dx\\
    \end{align*}
    Appliquer le changement de variable $x=\tan\left(\frac{t}{2}\right) $,
    \begin{align*}
    K&=\int_0^{\frac{\pi}{2}} \frac{\arctan\left(\frac{1}{2}\tan t\right)}{\sin t}\,dt
    \end{align*}
    On définit la fonction $H$ sur $\left[\frac{1}{2};1\right]$ par:
    \begin{align*}H(a)&=\int_0^{\frac{\pi}{2}} \frac{\arctan\left(a\tan t\right)}{\sin t}\,dt\end{align*}
    remarquer que $K=H\left(\dfrac{1}{2}\right)$ et,
    \begin{align*}H(1)&=\int_0^{\frac{\pi}{2}} \frac{t}{\sin t}\,dt\\
    &=\Big[t\ln\left(\tan\left(\frac{t}{2} \right)\right)\Big]_0^{\frac{\pi}{2}}-\int_0^{\frac{\pi}{2}}\ln\left(\tan\left(\frac{t}{2} \right)\right)\,dt\\
    &=-\int_0^{\frac{\pi}{2}}\ln\left(\tan\left(\frac{t}{2} \right)\right)\,dt\\
    \end{align*}
    Appliquer le changement de variable $x=\dfrac{t}{2}$,
    \begin{align*}H(1)&=-2\int_0^{\frac{\pi}{4}}\ln\left(\tan\left(t \right)\right)\,dt\\
    &=2\text{G}
    \end{align*}
    \begin{align*}H^\prime (a)&=\int_0^{\frac{\pi}{2}} \frac{\cos x}{1-(1-a^2)\sin^2 x}\,dt\\
    &=\left[\frac{1}{2\sqrt{1-a^2}}\ln\left(\frac{1+\sin(x)\sqrt{1-a^2}}{1-\sin(x)\sqrt{1-a^2}}\right)\right]_0^{\frac{\pi}{2}}\\
    &=\frac{1}{2\sqrt{1-a^2}}\ln\left(\frac{1+\sqrt{1-a^2}}{1-\sqrt{1-a^2}}\right)
    \end{align*}
    Donc,
    \begin{align*}H(1)-H\left(\frac{1}{2}\right)&=\int_{\frac{1}{2}}^1 \frac{1}{2\sqrt{1-a^2}}\ln\left(\frac{1+\sqrt{1-a^2}}{1-\sqrt{1-a^2}}\right)\,da\end{align*}Appliquer le changement de variable $y=\arctan\left(\sqrt{\dfrac{1+\sqrt{1-a^2}}{1-\sqrt{1-a^2}}}\right)$
    \begin{align*}H(1)-H\left(\frac{1}{2}\right)&=-2\int_{\frac{\pi}{12}}^{\frac{\pi}{4}} \ln\left(\tan y\right)\,dy\\
    &=-2\int_0^{\frac{\pi}{4}} \ln\left(\tan y\right)\,dy+2\int_0^{\frac{\pi}{12}} \ln\left(\tan y\right)\,dy
    \end{align*}
    Or, il est bien connu que,
    \begin{align*}\int_0^{\frac{\pi}{4}} \ln\left(\tan y\right)\,dy=-\text{G}\\\end{align*}
    De sorte que,
    \begin{align*}\int_0^{\frac{\pi}{12}} \ln\left(\tan y\right)\,dy=-\frac{1}{2}K\\\end{align*}
    Par ailleurs,
    \begin{align}\int_0^1 \frac{\arctan\left( \frac{x}{1-x^2}\right)}{x}\,dx-\int_0^1 \frac{\arctan x}{x}\,dx=\int_0^1 \frac{\arctan \left(x^3\right)}{x}\,dx\end{align}
    Dans la dernière intégrale appliquer le changement de variable $\displaystyle y=x^3$,
    \begin{align}\int_0^1 \frac{\arctan\left( \frac{x}{1-x^2}\right)}{x}\,dx-\int_0^1 \frac{\arctan x}{x}\,dx=\frac{1}{3}\int_0^1 \frac{\arctan x}{x}\,dx\end{align}
    Donc,
    \begin{align}\int_0^1 \frac{\arctan\left( \frac{x}{1-x^2}\right)}{x}\,dx&=\frac{1}{3}\int_0^1 \frac{\arctan x}{x}\,dx+\int_0^1 \frac{\arctan x}{x}\,dx\\
    &=\frac{4}{3}\int_0^1 \frac{\arctan x}{x}\,dx\\
    &=\frac{4}{3}\text{G}
    \end{align}
    Ainsi,
    \begin{align*}\int_0^{\frac{\pi}{12}} \ln\left(\tan y\right)\,dy&=-\frac{1}{2}\times \frac{4}{3}\text{G} \\
    &=\boxed{-\frac{2}{3}\text{G}}
    \end{align*}
  • avec une intégration par parties, tu trouves $\arctan(x)/x$ entre 0 et a et tu développes
    A demon  wind propelled me east of the sun
  • Bravo F.D.P ...
     
  • Gilles:

    Je sais comment développer en série entière la fonction $x\rightarrow\dfrac{\ln x}{1+x^2}$ dans un intervalle centré sur $x=0$.
    Mais tu fais quoi de la valeur $2-\sqrt{3}$? (même si elle a le bon goût d'être positive et plus petite que $1$)
  • $\alpha=2-\sqrt{3}=\tan\left(\frac{\pi}{12}\right)$.

    \begin{align}\int_0^\alpha \frac{\ln x}{1+x^2}dx&\overset{y=\frac{1}{\alpha}x}=\alpha\int_0^1 \frac{\ln(\alpha x)}{1+\alpha^2x^2}dx\\
    &=\arctan(\alpha)\ln\alpha+\alpha\int_0^1 \frac{\ln x}{1+\alpha^2x^2}dx\\
    &=\frac{\pi}{12}\ln\alpha+\frac{\alpha}{2}\int_0^1 \frac{\ln x}{1+i\alpha x}dx+\frac{\alpha}{2}\int_0^1 \frac{\ln x}{1-i\alpha x}dx\\
    &=\frac{\pi}{12}\ln\alpha+\frac{i}{2}\Big(\text{Li}_2\left(i\alpha\right)-\text{Li}_2\left(-i\alpha\right)\Big)
    \end{align}

    NB:
    $\displaystyle \text{Li}_2(a)=-a\int_0^1 \frac{\ln x}{1-ax}dx$ pour $|a|\leq 1$ complexe.
  • Je crois que FDP s'est inspiré de Sasha
    Le 😄 Farceur


  • Gebrane: Je ne connaissais pas ce message.
    L'évaluation de l'intégrale en fonction de valeurs du dilogarithme est aisée quand on a compris que la "bonne" formule pour le dilogarithme est celle que je donne plus haut. Cette formule est la "bonne" car on peut l'appliquer aisément à des arguments complexes de module plus petit que $1$. On n'a pas besoin d'analyse complexe, le logarithme est un brave logarithme franc du collier. B-)-

    NB: je ne m'intéresse qu'aux valeurs du dilogarithme que pour des arguments de module inférieur ou égal à $1$.
    Généralement dans les calculs d'intégrales on n'a pas besoin d'étendre la fonction dilogarithme à d'autres arguments.

    PS:
    Avec la définition "classique" du dilogarithme $\displaystyle \text{Li}_2(a)=-\int_0^a \dfrac{\ln(1-x)}{x}dx$ l'argument est une borne de l'intégrale. Si on veut considérer $a$ complexe non réel ce n'est pas cette formule qui va nous aider à définir le dilogarithme de $a$.
    La définition de base est $\displaystyle\text{Li}_2(a)=\sum_{n=1}^\infty \dfrac{a^n}{n^2}$ pour $|a|\leq 1$.
    Dans le calcul intégral on a rarement l'utilité de prolonger la fonction dilogarithme à d'autres arguments.
  • FDP je parlais de ce message http://www.les-mathematiques.net/phorum/read.php?4,2226350,2229284#msg-2229284 où tu fais intervenir $\int_0^{\frac{\pi}{12}} \ln\left(\tan y\right)\,dy$
    s'inspiré n'est pas un péché
    Le 😄 Farceur


  • Gebrane:
    Tu parles de cette égalité:
    $\displaystyle \int_0^{\frac{\pi}{12}}\ln(\tan x)dx=2\int_0^{\frac{\pi}{12}}\ln(\tan(3u))du$?

    J'ai indiqué où je l'ai trouvée. Representations of Catalan's constant (2001), David M. Bradley.
    Bradley indique qu'il la tient d'une autre personne mais je n'ai pas été capable, à ce jour, de trouver une source publiée antérieure. Bradley a écrit un autre papier où il utilise une généralisation de cette identité sur la fonction tangente si je me souviens bien.
  • La généralisation est celle-ci:

    $m=2n+1,n\geq 1$ un entier.
    $\displaystyle \dfrac{\tan(mx)}{\tan x}=\prod_{j=1}^n \tan\left(\frac{j\pi}{m}-x\right)\tan\left(\frac{j\pi}{m}+x\right)$

    Formule qu'on trouve dans A class of series acceleration formulae for Catalan's constant(2007), David M. Bradley.
  • Oui c'était cette égalité.
    Le 😄 Farceur


  • @Fdp: je ne vois pas trop le problème: $\int_0^a \dfrac{\ln t}{1 + t^2} \mathrm dt = \left[\arctan(t) \ln t\right]_0^a - \int_0^a \dfrac{\arctan t}{ t} \mathrm dt $ et $\dfrac{\arctan t}{ t} = \sum_{n \geq 0} (-1)^n \dfrac{t^{2n}}{2n+1} $; on intègre cette série entière entre 0 et $a$. On pouvait aussi développer directement $ \dfrac{1}{1 + t^2}$ et intervertir les sommes.

    En tout cas, bien vu (entre autre) pour l'emploi de l'expression du dilogarithme qui simplifie l'expression que mathematica m'avait donné.
    A demon  wind propelled me east of the sun
  • Dans le papier mentionné, il est indiqué le nom d'une personne Chris Hill, qui aurait inspiré Bradley pour ce calcul. Je n'arrive pas à identifier ce mathématicien pour lire ce qu'il a écrit. Mais il s'agit probablement du mathématicien qui a écrit cet article..
  • Gilles:

    Ce que je voulais dire est, comment évalues-tu : $\displaystyle \frac{i}{2}\Big(\text{Li}_2\left(i\alpha\right)-\text{Li}_2\left(-i\alpha\right)\Big)$ avec $\alpha=2-\sqrt{3}$?

    Ramanujan s'est intéressé à la fonction $\displaystyle F(x)=\int_0^x \frac{\arctan t}{t}dt$
    Je pense que dans le papier auquel je pense (qui est mentionné en référence dans le papier de Bradley) on trouve le calcul pour la valeur de $x$ qui nous intéresse.
  • moi, je n'avais pas obtenu cette expression (c'est le logiciel de calcul formel...); je m'étais limité à ce que j'ai écrit sans grand espoir d'en sortir vu que le résultat n'était pas dans Gradshteyn et Ryzhik.
    A demon  wind propelled me east of the sun
  • Voilà comment Ramanujan raisonne.
    Il définit la fonction $\displaystyle \phi(x)=\int_0^x \dfrac{\arctan t}{t}dt$

    Il donne la formule $\displaystyle \sum_{n=0}^\infty \dfrac{\sin(2(2n+1)x)}{(2n+1)^2}=\phi(\tan x)-x\ln(\tan x)$
    Valable pour $0\leq x<\dfrac{\pi}{2}$. Formule qu'il dit pouvoir être démontrée par différentiation.

    (c'est extrait du papier de 1915 consacré à la fonction $\displaystyle \int_0^x \dfrac{\arctan t}{t}dt$)
  • Pas bien compris le reglement du club. Voici une integrale connue des probabilistes, mais dont une demonstration directe serait la bienvenue :


    $$\int_0^1\frac{\sin (\pi x)}{x^x(1-x)^{1-x}}dx=\frac{\pi}{e}.$$
  • Bonjour P et bienvenue au club (tu), Cherches-tu une méthode sans l'analyse complexe?

    Le but de ce club était pour moi, un club d’échanges et d’apprentissages.
    Des solutions détaillées lorsque on trouve une méthode, étaient souhaités:
    Apprentissage par l'exemple. Donc le but, c'est d'en profiter les autres
    comme le fait systématiquement FDP . Et surtout ne pas noyer le fil par plusieurs calculs à la fois: traiter un par un!
    Le 😄 Farceur


  • La revue American mathematical monthly a osé publier, semble-t-il, dans son dernier numéro, le problème suivant:
    Montrer que $\displaystyle \int_0^1 \frac{\ln(1-x)\ln(1+x)}{x}dx=-\frac{5}{8}\zeta(3)$ (problème 12256).

    Je suis bien certain que si on cherche cette intégrale sur MathExchange on la trouve.

    PS:
    C'est confirmé:
    https://math.stackexchange.com/questions/3085699/evaluate-int-01-frac-ln-1-x-ln-1-xx-dx
  • Finalement j’ai mis mon intégrale dans un nouveau post.
  • FDP, l’intégrale de P est de même style que la jolie intégrale de dedekind93. Il se fait avec l'analyse complexe, mais je ne sais rien s'il se fait par l'analyse réel
    Le 😄 Farceur


  • \begin{align}J&=\int_{0}^{1} \frac{\arctan(x) - \arctan(x^2)}{1-x} dx\\
    A&=\int_0^1 \frac{2x\ln(1-x)}{1+x^4}dx,B=\int_0^1 \frac{2x\ln(1+x)}{1+x^4}dx\\
    A+B&=\int_0^1 \frac{x\ln(1-x^2)}{1+x^4}dx\\
    &\overset{u=x^2}=\underbrace{\int_0^1 \frac{\ln(1-u)}{1+u^2}du}_{y=\frac{1-u}{1+u}}\\
    &=\underbrace{\int_0^1 \frac{\ln\left(\frac{2}{1+y}\right)}{1+y^2}dy}_{z=\frac{1-y}{1+y}}+\underbrace{\int_0^1 \frac{\ln y}{1+y^2}dy}_{=-\text{G}}\\
    &=\underbrace{\int_0^1 \frac{\ln(1+z)}{1+z^2}dz}_{=\frac{1}{8}\pi\ln 2}-\text{G}\\
    A-B&=\int_0^1 \frac{2x\ln\left(\frac{1-x}{1+x}\right)}{1+x^4}dx\\
    &\overset{y=\frac{1-y}{1+y}}=2\int_0^1 \frac{(1-y^2)\ln y}{y^4+6y^2+1}dy\\
    &\overset{\text{IPP}}=\left[\arctan\left(\frac{2y}{1+y^2}\right)\ln y\right]_0^1-\int_0^1 \frac{\arctan\left(\frac{2y}{1+y^2}\right)}{y}dy\\
    &=-\int_0^1 \frac{\arctan\left(\frac{2y}{1+y^2}\right)}{y}dy\\
    &\overset{y=\tan t}=-2\int_0^{\frac{\pi}{4}}\frac{\arctan\left(\sin(2t)\right)}{\sin(2t)}dt\\
    &\overset{y=\sin(2t)}=-\int_0^1\frac{\arctan y}{y\sqrt{1-y^2}}dy\\
    &=-\int_0^1 \int_0^1\frac{1}{(1+x^2y^2)\sqrt{1-y^2}}dxdy\\
    &=-\int_0^1 \left[\frac{\arctan\left(y\sqrt{\frac{1+x^2}{1-y^2}}\right)}{\sqrt{1+x^2}}\right]_0^1 dx\\
    &=-\frac{1}{2}\pi\int_0^1 \frac{1}{\sqrt{1+x^2}}dx\\
    &=-\frac{1}{2}\pi\cdot\text{arcsinh}(1)\\
    J&\overset{\text{IPP}}=\underbrace{\int_0^1 \frac{\ln(1-x)}{1+x^2}dx}_{=A+B}-2\int_0^1 \frac{x\ln(1-x)}{1+x^4}dx\\
    &=(A+B)-A\\
    &=B\\
    &=\frac{1}{2}(A+B)-\frac{1}{2}(A-B)\\
    &=\boxed{\dfrac{1}{16}\pi\ln 2-\dfrac{1}{2}\text{G}+\dfrac{1}{4}\pi\cdot\text{arcsinh}(1)}
    \end{align}
  • Bonjour,

    $\displaystyle argsinh(1)=\ln(1+\sqrt{2}).$
  • @Fin de partie ta solution est une œuvre d’art merci
  • Bonsoir

    Trouver une forme explicite de $I_n=\int_0^{+\infty} \frac {\ln(x^n+1)}{x^n+1} dx$ un oubli n>1
    Je vais la donner à la fin de ce mois si non trouvé pour s'assurer que je ne trolle pas
    Idée d'une preuve,
    1) montrer que l’intégrale converge
    2) poser F(t)=$\int_0^{+\infty} \frac {\ln(t^nx^n+1)}{x^n+1} dx$ et calculer F'(t).
    3) très difficile, montrer que $\forall t\neq 1$ et $ t>0$, on a edit faute de frappe corrigée ( merci bd) $F'(t)=\frac{\pi}{\sin(\frac {\pi}n)} \frac {t^{n-2}(1-t)}{1-t^n} dx$
    4) Calculer $I_n$

    Ça vous rappelle peut être l’intégrale effrayante $\int_0^{1} \frac {\ln(x^3+1)}{x^3+1} dx$
    Le 😄 Farceur


  • bonjour, disons $n \geq 2$ sinon la convergence est compromise.:-)
    A demon  wind propelled me east of the sun
Connectez-vous ou Inscrivez-vous pour répondre.